An equation invariant under change of variable

In summary: We can start with the first term and look at the second derivative:$$\frac{dv}{d\alpha} = \frac{dv(\alpha)}{d\alpha} = \frac{dv(\alpha)}{d\theta}\frac{d\theta}{d\alpha} = \frac{du(\theta)}{d\theta}(-1) = -\frac{du}{d\theta} $$And, if we take the second derivative, we get another factor of ##-1##, which cancels out and we find that:$$\frac{d^2v}{d\alpha^2} = \frac{d^2u}{d\theta
  • #1
Kashmir
465
74
It's said that the below equation is invariant under a substitution of ##-\theta## for ##\theta## ,

##\frac{d^{2} u}{d \theta^{2}}+u=-\frac{m}{l^{2}} \frac{d}{d u} V\left(\frac{1}{u}\right)##

I can't understand this how this is so. It's supposed to be obvious but I can't see it.

Please help. Thank you
 
Physics news on Phys.org
  • #2
Kashmir said:
It's said that the below equation is invariant under a substitution of ##-\theta## for ##\theta## ,

##\frac{d^{2} u}{d \theta^{2}}+u=-\frac{m}{l^{2}} \frac{d}{d u} V\left(\frac{1}{u}\right)##

I can't understand this how this is so. It's supposed to be obvious but I can't see it.

Please help. Thank you
You have to ask yourself what does "the equation is invariant under a substitution of ##-\theta## for ##\theta##" actually mean? Unless you know what that means, how can you see it?
 
  • Like
Likes Kashmir
  • #3
PeroK said:
You have to ask yourself what does "the equation is invariant under a substitution of ##-\theta## for ##\theta##" actually mean? Unless you know what that means, how can you see it?
I don't know. I tried searching on the internet but didn't find it. Can you tell me what that means?
 
  • #4
Kashmir said:
I don't know. I tried searching on the internet but didn't find it. Can you tell me what that means?
Let's look at the general case. We have a function ##u(\theta)## which satisfies a differential equation. We then look at a substitution, which in general is of the form ##\theta = f(\alpha)## or ##\alpha = g(\theta)##. Typical examples might be ##\theta = a\alpha + b## or ##\alpha = \cos(\theta)##. In this case we have the substitution ##\alpha = -\theta##.

Note that if you don't "see" what's happening, then it can be confusing to try to work with variables ##\theta## and ##-\theta##, so it's conceptually clearer to let ##\alpha = -\theta##.

A second point is that physicists especially tend to use the same function for both the original ##u(\theta)## and the new ##u(\alpha)##. This is a notational shortcut, but in fact we really want to look at a new function, which we'll call ##v##, where: $$v(\alpha) = u(\theta) = u(f(\alpha))$$ This shows that ##v## is not the same mathematically defined function as ##u##, but the composition of ##u \circ f##. That said, it's seen by physicists as the same "physical" function, so usually they don't bother with this subtlety.

Then, we have to look for the equation that is satisfied by the function ##v(\alpha)## - and it may or may not take the same form as the original equation. Often these substitutions are used to try to change the equation to one that is easier to solve. In this case, however, it is to show that the system has some sort of symmetry in the variable ##\theta##.

The full, rigorous way to tackle this, therefore, is to use the substitution ##\alpha = -\theta## and see what becomes of the original equation. We can start with the first term and look at the second derivative:
$$\frac{dv}{d\alpha} = \frac{dv(\alpha)}{d\alpha} = \frac{dv(\alpha)}{d\theta}\frac{d\theta}{d\alpha} = \frac{du(\theta)}{d\theta}(-1) = -\frac{du}{d\theta} $$
And, if we take the second derivative, we get another factor of ##-1##, which cancels out and we find that:
$$\frac{d^2v}{d\alpha^2} = \frac{d^2u}{d\theta^2} $$
I'll leave you to work on the terms on the RHS.

As I said, this is the full rigorous way to see what's happening. It's certainly quicker just to say that it's obvious!
 
  • Like
Likes Delta2 and Kashmir
  • #5
PeroK said:
Let's look at the general case. We have a function ##u(\theta)## which satisfies a differential equation. We then look at a substitution, which in general is of the form ##\theta = f(\alpha)## or ##\alpha = g(\theta)##. Typical examples might be ##\theta = a\alpha + b## or ##\alpha = \cos(\theta)##. In this case we have the substitution ##\alpha = -\theta##.

Note that if you don't "see" what's happening, then it can be confusing to try to work with variables ##\theta## and ##-\theta##, so it's conceptually clearer to let ##\alpha = -\theta##.

A second point is that physicists especially tend to use the same function for both the original ##u(\theta)## and the new ##u(\alpha)##. This is a notational shortcut, but in fact we really want to look at a new function, which we'll call ##v##, where: $$v(\alpha) = u(\theta) = u(f(\alpha))$$ This shows that ##v## is not the same mathematically defined function as ##u##, but the composition of ##u \circ f##. That said, it's seen by physicists as the same "physical" function, so usually they don't bother with this subtlety.

Then, we have to look for the equation that is satisfied by the function ##v(\alpha)## - and it may or may not take the same form as the original equation. Often these substitutions are used to try to change the equation to one that is easier to solve. In this case, however, it is to show that the system has some sort of symmetry in the variable ##\theta##.

The full, rigorous way to tackle this, therefore, is to use the substitution ##\alpha = -\theta## and see what becomes of the original equation. We can start with the first term and look at the second derivative:
$$\frac{dv}{d\alpha} = \frac{dv(\alpha)}{d\alpha} = \frac{dv(\alpha)}{d\theta}\frac{d\theta}{d\alpha} = \frac{du(\theta)}{d\theta}(-1) = -\frac{du}{d\theta} $$
And, if we take the second derivative, we get another factor of ##-1##, which cancels out and we find that:
$$\frac{d^2v}{d\alpha^2} = \frac{d^2u}{d\theta^2} $$
I'll leave you to work on the terms on the RHS.

As I said, this is the full rigorous way to see what's happening. It's certainly quicker just to say that it's obvious!
Thank you so much. I'll work out all the terms and see what I end up with.
Thank you again. :)
 
  • #6
PeroK said:
Let's look at the general case. We have a function ##u(\theta)## which satisfies a differential equation. We then look at a substitution, which in general is of the form ##\theta = f(\alpha)## or ##\alpha = g(\theta)##. Typical examples might be ##\theta = a\alpha + b## or ##\alpha = \cos(\theta)##. In this case we have the substitution ##\alpha = -\theta##.

Note that if you don't "see" what's happening, then it can be confusing to try to work with variables ##\theta## and ##-\theta##, so it's conceptually clearer to let ##\alpha = -\theta##.

A second point is that physicists especially tend to use the same function for both the original ##u(\theta)## and the new ##u(\alpha)##. This is a notational shortcut, but in fact we really want to look at a new function, which we'll call ##v##, where: $$v(\alpha) = u(\theta) = u(f(\alpha))$$ This shows that ##v## is not the same mathematically defined function as ##u##, but the composition of ##u \circ f##. That said, it's seen by physicists as the same "physical" function, so usually they don't bother with this subtlety.

Then, we have to look for the equation that is satisfied by the function ##v(\alpha)## - and it may or may not take the same form as the original equation. Often these substitutions are used to try to change the equation to one that is easier to solve. In this case, however, it is to show that the system has some sort of symmetry in the variable ##\theta##.

The full, rigorous way to tackle this, therefore, is to use the substitution ##\alpha = -\theta## and see what becomes of the original equation. We can start with the first term and look at the second derivative:
$$\frac{dv}{d\alpha} = \frac{dv(\alpha)}{d\alpha} = \frac{dv(\alpha)}{d\theta}\frac{d\theta}{d\alpha} = \frac{du(\theta)}{d\theta}(-1) = -\frac{du}{d\theta} $$
And, if we take the second derivative, we get another factor of ##-1##, which cancels out and we find that:
$$\frac{d^2v}{d\alpha^2} = \frac{d^2u}{d\theta^2} $$
I'll leave you to work on the terms on the RHS.

As I said, this is the full rigorous way to see what's happening. It's certainly quicker just to say that it's obvious!
The rhs converts to :
##\frac{m}{l^{2}} \frac{d}{d v(-\theta)} V\left(\frac{1}{v(-\theta)}\right)##

And the total equation can be written as ##\frac{d^{2}v(-\theta) }{d(-\theta)^{2}}+v(-\theta)=-\frac{m}{l^{2}} \frac{d}{d v(-\theta)} V\left(\frac{1}{v(-\theta)}\right)## And noting that ##u(\theta)=v(-\theta)## we Finally have
##\frac{d^{2}}{d(-\theta)^{2}} u(\theta)+u(\theta)=-\frac{m}{l^{2}} \frac{d}{d u} V\left(\frac{1}{u}\right)##

Is this correct then?
 
  • #7
Kashmir said:
we Finally have
##\frac{d^{2}}{d(-\theta)^{2}} u(\theta)+u(\theta)=-\frac{m}{l^{2}} \frac{d}{d u} V\left(\frac{1}{u}\right)##

Is this correct then?
It's true that if you replace ##\theta## by ##-\theta## in the first term that the equation still holds. But, you were supposed to check that the whole equation was still valid with ##\theta## replaced by ##-\theta##.

Whether you use ##u(-\theta)## or ##v(-\theta)##, you need to end up with ##-\theta## everywhere.

That's why I said to be careful not to confuse ##\theta## with ##-\theta##. If the substitution was ##\alpha = \cos \theta##, you clearly need to end up with ##\alpha## or ##\cos \theta## in the equation. You couldn't just leave it as ##\theta## in that case. Do you see that?
 
  • #8
PeroK said:
It's true that if you replace ##\theta## by ##-\theta## in the first term that the equation still holds. But, you were supposed to check that the whole equation was still valid with ##\theta## replaced by ##-\theta##.

Whether you use ##u(-\theta)## or ##v(-\theta)##, you need to end up with ##-\theta## everywhere.

That's why I said to be careful not to confuse ##\theta## with ##-\theta##. If the substitution was ##\alpha = \cos \theta##, you clearly need to end up with ##\alpha## or ##\cos \theta## in the equation. You couldn't just leave it as ##\theta## in that case. Do you see that?
I'm sorry for taking up your time but I did not simply replace ##\theta## by ##-\theta##. I used the equations you proved and the one I proved as the RHS then I proceeded on. Should I add all of my steps if that makes it clear?
 
  • #9
Kashmir said:
I'm sorry for taking up your time but I did not simply replace ##\theta## by ##-\theta##. I used the equations you proved and the one I proved as the RHS then I proceeded on. Should I add all of my steps if that makes it clear?
I know, but in the end you quoted an equation with mostly ##\theta## as the variable and not ##-\theta##. If you stick with ##\alpha## and put a note after the equation that says "where ##\alpha = -\theta##" you can't go wrong. Note that ##-\theta \neq \theta##.
 
  • Like
Likes Kashmir
  • #10
PeroK said:
I know, but in the end you quoted an equation with mostly ##\theta## as the variable and not ##-\theta##. If you stick with ##\alpha## and put a note after the equation that says "where ##\alpha = -\theta##" you can't go wrong. Note that ##-\theta \neq \theta##.
So have i ended up wrong with the final result?
 
  • #11
Kashmir said:
So have i ended up wrong with the final result?
Yes. For example:
$$\frac{d\sin(-\theta)}{d(-\theta)} \neq \frac{d\sin(\theta)}{d(-\theta)}$$
 
  • #12
PeroK said:
Yes. For example:
$$\frac{d\sin(-\theta)}{d(-\theta)} \neq \frac{d\sin(\theta)}{d(-\theta)}$$
I'll Redo it.
 
  • #13
Kashmir said:
I'll Redo it.
You don't have to. This was correct:

Kashmir said:
And the total equation can be written as ##\frac{d^{2}v(-\theta) }{d(-\theta)^{2}}+v(-\theta)=-\frac{m}{l^{2}} \frac{d}{d v(-\theta)} V\left(\frac{1}{v(-\theta)}\right)##
A physicist would simply replace ##v## by ##u## there, but you have to leave the ##-\theta## everywhere, because that was the substitution after all!
 
  • Like
Likes Kashmir
  • #14
PeroK said:
You don't have to. This was correct:A physicist would simply replace ##v## by ##u## there, but you have to leave the ##-\theta## everywhere, because that was the substitution after all!
So My final result is ok?
 
  • #15
Kashmir said:
So My final result is ok?
No. Your final result is wrong. Your intermediate result was correct.
 
  • Like
Likes Kashmir
  • #16
By intermediate you mean ##\frac{d^{2}v(-\theta) }{d(-\theta)^{2}}+v(-\theta)=-\frac{m}{l^{2}} \frac{d}{d v(-\theta)} V\left(\frac{1}{v(-\theta)}\right)##
 
  • #17
Kashmir said:
By intermediate you mean ##\frac{d^{2}v(-\theta) }{d(-\theta)^{2}}+v(-\theta)=-\frac{m}{l^{2}} \frac{d}{d v(-\theta)} V\left(\frac{1}{v(-\theta)}\right)##
Yes.
 
  • #18
PeroK said:
You don't have to. This was correct:A physicist would simply replace ##v## by ##u## there, but you have to leave the ##-\theta## everywhere, because that was the substitution after all!
isn't that what I did in the final equation? Just replaced u by v everywhere and got ##\frac{d^{2}}{d(-\theta)^{2}} u(\theta)+u(\theta)=-\frac{m}{l^{2}} \frac{d}{d u} V\left(\frac{1}{u}\right)## ?
 
  • #19
Kashmir said:
isn't that what I did in the final equation? Just replaced u by v everywhere and got ##\frac{d^{2}}{d(-\theta)^{2}} u(\theta)+u(\theta)=-\frac{m}{l^{2}} \frac{d}{d u} V\left(\frac{1}{u}\right)## ?
No, because you also replaced ##-\theta## with ##\theta##. A physicist wouldn't use ##v## at all, but just use ##u## for both functions - it saves having to think of a new letter!

And, as long as you are careful it works out. It just doesn't quite stand up to mathematical scutiny. Some physics books mention it and say "technically we should introduce a new function here, but ..." and other books don't say anything.

It's useful to know that there are two functions involved, even if you choose to use the same letter for both.
 
  • #20
PeroK said:
No, because you also replaced ##-\theta## with ##\theta##. A physicist wouldn't use ##v## at all, but just use ##u## for both functions - it saves having to think of a new letter!

And, as long as you are careful it works out. It just doesn't quite stand up to mathematical scutiny. Some physics books mention it and say "technically we should introduce a new function here, but ..." and other books don't say anything.

It's useful to know that there are two functions involved, even if you choose to use the same letter for both.
I'll study all your replies once again. Thank you so much :)
 
  • #21
If this is correct :##\frac{d^{2}v(-\theta) }{d(-\theta)^{2}}+v(-\theta)=-\frac{m}{l^{2}} \frac{d}{d v(-\theta)} V\left(\frac{1}{v(-\theta)}\right)##

And also we started as ##u(\theta)=v(-\theta)##

Then if I just use it in the first equation I will get ##\frac{d^{2}}{d(-\theta)^{2}} u(\theta)+u(\theta)=-\frac{m}{l^{2}} \frac{d}{d u} V\left(\frac{1}{u}\right)## but you said its wrong.
I can't see what went wrong?
 
  • #22
Kashmir said:
Then if I just use it in the first equation I will get ##\frac{d^{2}}{d(-\theta)^{2}} u(\theta)+u(\theta)=-\frac{m}{l^{2}} \frac{d}{d u} V\left(\frac{1}{u}\right)## but you said its wrong.
I can't see what went wrong?
It's not wrong, but it's not what you were asked to show. You were asked to carry out the substitution ##\theta \rightarrow -\theta##.

I cannot stress enough (and this is the third time I've told you) that you must use a different variable ##\alpha = -\theta## to resolve these sort of conceptual problems.

The question then asks you to carry out the substitition ##\alpha = -\theta##. Your final answer must involve ##\alpha## and only ##\alpha##. It can no longer have ##\theta## in it.

If someone asked you to translate "I don't know" into French, then a correct answer is "je ne sais pas". If you translate it back and give your answer as "I don't know", then of course that means the same thing, but it's no longer in French - it's back into English again - and you were asked to give an answer in French.
 

What is an equation invariant under change of variable?

An equation invariant under change of variable is an equation that remains unchanged when the variables within the equation are replaced with new variables. This means that the solution to the equation will also remain the same regardless of the variables used.

Why is it important to have equations invariant under change of variable?

Having equations invariant under change of variable allows for greater flexibility and ease in solving mathematical problems. It also allows for the same equation to be applied to different scenarios without needing to change the equation itself.

How can you determine if an equation is invariant under change of variable?

To determine if an equation is invariant under change of variable, you can perform a substitution of variables and see if the equation remains unchanged. If the equation does not change, it is invariant under change of variable.

What are some examples of equations that are invariant under change of variable?

Some examples of equations that are invariant under change of variable include the Pythagorean theorem, the quadratic formula, and the equation for calculating the area of a circle. These equations remain the same regardless of the variables used.

How is invariance under change of variable related to symmetry?

Invariance under change of variable is related to symmetry because it means that the equation remains unchanged under different transformations. This is similar to how symmetry means that an object remains unchanged under different transformations such as rotations or reflections.

Similar threads

  • Differential Equations
Replies
1
Views
741
  • Differential Equations
Replies
1
Views
1K
  • Differential Equations
Replies
1
Views
2K
Replies
4
Views
1K
Replies
2
Views
1K
  • Differential Equations
Replies
3
Views
1K
  • Differential Equations
Replies
1
Views
2K
  • Differential Equations
Replies
6
Views
2K
  • Differential Equations
Replies
8
Views
2K
  • Differential Equations
Replies
4
Views
1K
Back
Top